Download as pdf
Download as pdf
You are on page 1of 133
The _ Book of Map Growth Reading fas) ©YBtong Reading Questions for the MAP Exam® 8" Grade Informational Text - Basic 10 Questions Quiz1 Directions: Identify the choice that best completes the statement or answers the question. Read the following passage. DNA sequencing has given us a map of genetic connections between peoples, but even before that, anthropologists had found a link between people from Asia and Native Americans. They both share several distinctive features in their bite, or the shape and pattern of their teeth. That led to suggestions that Native Americans must—thousands of years ago—have been able to cross from one continent to the other. Geologists considered it feasible, because in the last Ice Age some 70- 100,000 years ago, sea levels were much lower than they are today: millions of cubic meters of water were locked up in ice sheets. That meant that for 30,000 years it was possible to walk across a land bridge they called “Beringia” from Siberia to Alaska. It was a 750 mile tract of grassland, with mild summers, and dry, virtually snow-free winters instead of a dangerous shipping lane called the Bering Strait. Now, the sequencing of the human genome confirms the hypothesis: people from Russia share the same genes as Native Americans. In some cases there may be physical similarities, and these are explained by the fact that thousands of years ago, some intrepid explorers made the long trek east and south to populate what is now the United States. 1. Prior to DNA testing, what clue suggested a relationship between Native Americans and Asians? ‘A. genetic connections 8. dental evidence C. geological records D. similar appearance 2. The author would probably disagree with which of these statements? A. The settlement of North America via the Bering land bridge is a remarkable achievernent given the primitive tools available 70,000 years ago. B, The settlement of new lands during the ice age was inevitable given mankind’s natural tendency to explore. C. The existence of Beringia has yet to be proven since the area between Sibe not submerged under the Bering Sea. D. The crossing from Asia to America would have been a gradual dispersal of peoples rather than a definite migration. and Alaska is 3. As itis used in this passage, the word “tract” most nearly means: A. area 8. strip C. path D. arrangement Read the following two passages. Passage 1 In 2013, the University of Vermont took a stand on plastic. They banned the sale of bottled water on campus. Many other places—including entire states—followed suit. The intention was to reduce the amount of plastic getting into landfills or ending up in oceans, since 70% of plastic water bottles are not recycled. Unfortunately, the ban had unforeseen consequences. The sale of higher calorie drinks went up. Those between 10-50 calories went up by 12%, juice sales increased by 111%, and sugary drinks went up by 10%. In addition, many of the additional drinks were carbonated and in plastic bottles using three times the amount of plastic (to withstand the pressure of the gas). In U.S. landfills, carbonated bottles account for 13%, while the thinner, lighter water bottles account for just 3%. At the same time as bans on the sale of water in single-use bottles increased, ‘the Department of Homeland Security advised Americans to prepare for natural and man-made disasters by buying bottled water. Passage 2 In the U.S. 67.3% of bottled water comes in single-use plastic bottles, and the majority of us fail to dispose of them properly. It’s a matter of concern as the Ellen MacArthur Foundation estimates that by 2050, there will be more plastic (by weight) in the oceans than there are fish. It’s a problem, because the plastic is produced from crude oil—a non-renewable source, heavy with pollutants— and it breaks down in sunlight. Floating about in an ocean, a plastic bottle can break up into millions of tiny pieces in as little as a year. Those pieces attract a coating of algae, and that makes them look and smell like food. Fish, turtles, whales, and other marine mammals all end up munching micro plastics, and while most remains in the animals’ guts and is expelled as waste, tiny amounts can accumulate in tissues and ends up on our dinner plates. 4, In Passage 1, which is the best evidence that the University of Vermont's strategy backfired? ‘A. “Many other places—including entire states—followed suit.” B, “The sale of higher calorie drinks went up.” C. “Many of the additional drinks were carbonated, and in plastic bottles using three times the amount of plastic.” D. “The Department of Homeland Security advised Americans to prepare for natural and man- made disasters by buying bottled water.” 5. The author of Passage 2 suggests that most single-use plastic bottles: ‘A. end up in the ocean. B. are unnecessary. C. could be replaced D. are not recycled, 6. What conclusion can be drawn from Passage 2? ‘A. We should use less bottled water. B, Plastic bottles should be biodegradable. C. Micro plastics are a hazard to shipping. D. Plastics in the food chain will end up in us. 7. Taken together, the passages imply that single-use plastic bottles should be: A. avoided. 8. replaced. C. banned. D. recycled. Read the following passage Beowulf was made into a film in 2007, but the story goes back much further than that. Itis an epic poem about a Scandinavian warrior of the same name, and first appeared in print in 1815. It is written in Anglo-Saxon—a language spoken in Britain from around 450 CE to about 1150 CE— on a single manuscript that is 1,000 years old. However, experts believe the manuscript is a copy, and that the original story dates from 700 CE—closer to Roman Britain than the Norman Conquest. It is set still farther back in time, in around 450 CE, and follows the exploits of Beowulf and his band of men. There isn’t any evidence of a real-life Beowulf, but he may have been based on a Scandinavian warrior or folk hero; certainly some of the events and places in the story are real and can be verified. It is one of those rare tales that stands the test of time, in the same way Medea by Euripides is still a good story, or Homer's Odyssey is worth reading, or we can still get the point of Chaucer's Canterbury Tales. A good story is a good story, and will keep being retold so long as people are still motivated by the same things: love, faith, power—or all three. 8. According to the passage, Beowulf was probably originally written: ‘A. in Roman Britain in around 450 CE 8, in the Dark Ages of around 700 CE C. during the Normal Conquest of 1066. D. for the first time in the 19" century. 9. Which choice best sums up the author's opinion of Beowulf? ‘A. He was likely a real person because of the Scandinavian warrior tradition. B He was a figment of the original author's imagination. C. He was probably based on a real person since other parts of the story are true. D. He was a real warrior known at the time the story was written. 10. The author compares Beowulf to Medea, the Odyssey, and the Canterbury Tales in order to suggest that: ‘A. human nature doesn’t change 8, each is roughly the same date. C. good literature will always last. D. the underlying plots are the same. ANSWERS Informational Text - Basic Quiz 1 The passage says: “They both share several distinctive features in their bite, or the shape and pattern of their teeth,” meaning that it was dental evidence that first gave a clue as to the connection between Native Americans and Asians. ‘The author would agree with all the statements except this one. Although physical evidence is now buried under the Bering Sea, “the sequencing of the human genome confirms the hypothesis,” so there must have been a land bridge (which just happens to be called Beringia after the Bering Sea under which it is now submersed). A tract isa large expanse of land. The University’s aim was to “reduce the amount of plastic getting into landfills or ending up in oceans,” but when they banned single-use plastic water bottles, many people turned to carbonated drinks, which use “three times the amount of plastic” and since they “account for 13%” of landfills (compared to 3%), the strategy had the opposite effect to the one they wanted. The author says: “the majority of us fail to dispose of them [single-use plastic water bottles] properly,” meaning that we should recycle them, but we don’t. The passage leads from plastic bottles not being recycled, to plastic bottle sin the ocean, to photo-degraded plastics in the ocean, to plastics looking like food and getting into the food chain and that “ends up on our dinner plates.” The implication is that ultimately, it will end up inside us, too. Taken as a whole, the passages suggest that we need to find something we can replace single-use plastic bottles with. (A) is too mild, as “avoid” doesn’t reflect how big a problem single-use plastic bottles are. (C) goes too far, because banning single- use plastic water bottles, unless they are replaced means banning water too. (0) is not implied, because both passages say—even if we know we should—we don’t recycle. Passage 1 says "70% are not recycled,” and Passage 2 says “the majority of us fal to dispose of them [single-use plastic bottles] properly.” The passage says: “the original story dates from 700 CE.” The author says that although there isn’t any evidence of a real-life Beowulf, “he may have been based on a Scandinavian warrior or folk hero,” and then reinforces this idea by saying “certainly some of the events and places in the story are real and can be verified.” 10. A The author refers to famous stories from the past—Medea by Euripides, The Odyssey by Homer, and the Canterbury Tales by Chaucer —to suggest that people don’t change. If “people are still motivated by the same things: love, faith, power—or all three,” then they will still find the same stories interesting today—because “a good story is a good story.” ©YBtong Reading Questions for the MAP Exam® 8" Grade Informational Text - Basic 10 Questions Quiz 2 Directions: Identify the choice that best completes the statement or answers the question. Read the following passage. Sydney Opera House is one of the most recognized buildings in the world. Its great arching roofs are designed to look like the sails of a ship moored at Bennelong Point, a promontory jutting out into Sydney Harbor. However, at the time it was built, computer-aided design was in its infancy, and steel and concrete construction at the limits of technology. The design came from the drawing board of Jorn Utzon of Denmark, a little known architect on the global stage, who won a competition for the project in 1956. He wanted the Opera House to be more like a sculpture than a building, but that involved finding a way to create the 200-foot high sails so that the structure shifted with each perspective. It took until 1963 to come up with a way of producing the concrete shells and their steel-reinforced ribs, with the engineers and architect working together. What they ‘came up with was a mold in the shape of an arc ofa circle. By casting arcs of different lengths, they were able to produce ribs from the smallest to the biggest economically. Even so, the four-year, 7 million dollar project ended up taking 14 years and cost 102 million dollars (Australian dollars). 1. The author characterizes the design of the Sydney Opera House as: A. sculpturally impressive. 8. typically Scandinavian C. technically ambitious. D. practically impossible 2. What is meant by “the structure shifted with each perspective”? ‘A. The giant sails turned the building slowly round. B, The design revealed a different aspect from every angle. C. The central stage could be moved to face a different direction. D. The 200-foot high sails could be shifted to alter the look of the building, 3. What effect does the author’s use of the word economically have? A. Itssuggests that any other method would have been too expensive. B, It implies that cost was a consideration in the building's design. C. It explains why the building went so massively over-budget. D. It indicates the financial pressures facing the engineers. Read the following interview and passage. Passage 1: Interviewer: So, Chris. Current thinking is that there were occasional forays out of Africa, but Homo sapiens didn’t make it very far—not really beyond the eastern Mediterranean—until 60,000 years ago, and that that’s when they began spreading east into Asia and West into Europe. I say current thinking, but actually ‘the new analysis on these skulls from Greece is changing that, right? Scientist: Yes. The skulls were found in the 1970s and thought to be about 50,000 years old, but new technology enabled us to test the material and get accurate dates. One skull was Neanderthal and dated to 170,000 years ago, but the real surprise was that the other skull was Homo sapiens—and it was older. Interviewer: Older than 170,000 years? Scientist: Substantially older. It is at least 210,000 years old, which suggests that at least some of our ancestors made it to Europe much, much earlier than suspected. It ‘means that several other finds will need to be re-evaluated in the light of this. We may have to come up with an entirely different picture of early modern man: Passage 2 In the 1980s DNA analysis seemed to confirm that Homo sapiens evolved in Africa, probably around 200,000 years ago. Based on the fossil record, the “Out of Africa” mass migration theory suggested modern humans went first to Asia 60—80,000 years ago, to Australia via Papua New Guinea 50,000 years ago, and to Europe 40,000 years ago. Finally, around 15,000 years ago, humans crossed from Asia to North America and from there to South America, However, experts acknowledge that the dispersal of modern humans in small numbers had been going on for thousands of years before that. There is evidence of H.sapiens in Israel 170,000 years ago, for example, alongside Neanderthals. However, the re-dating of finds from a cave in Greece, one Homo sapien’s skull and one Neanderthal skull, shows that, astonishingly, modern man was present in Europe 210,000 years ago—170,000 years earlier than previously thought. Their presence might explain enigmatic DNA evidence suggesting that interbreeding between Neanderthal and H. sapiens populations was going on much earlier than the “Out of Africa” migration would allow. 4, According to the interview, what is the significance of the new evidence? ‘A. Previous theories about the first Homo sapiens need to be revisited. 8. Homo sapiens arrived in Europe before Neanderthals. C. Everything we thought we knew about human ancestors is incorrect. D. Homo sapiens reached Europe before they reached Asia. 5. In the interview, the scientist's comment “substantially older” is an example of: A. hyperbole. 8B. confirmation, C. clarification, D. hedging. 6. The main idea of Passage 2 is that Homo sapiens: A. did not leave Africa in the mass migration suggested by the “Out of Africa” theory. B. evolved in Europe 210,000 years ago, alongside Neanderthals, as evidenced by the Spanish skulls. C. reached Europe 210,000 years ago, explaining DNA evidence of Neanderthal interbreeding. D. migrated from Africa on multiple occasions with one wave reaching Europe 210,000 years ago. 7. In presenting the evidence of Homo sapien’s presence in Europe much earlier than previously thought, the interview has all the following advantages over the written text EXCEPT: A. The tone of voice of both interviewer and expert can reveal their feelings. 8B, The interviewer can seek clarification from the expert through questioning, C. The expert can confirm suggestions offered by the interviewer. D. The main idea can be clearly pointed out for a lay audience, Read the following passage The jetstream is a moving ribbon of air high above the Earth (about 7 miles high) in the upper troposphere that runs roughly west to east. It isn’t constant like a streamer from a fan, but meanders like a river, first one way and then the next, but at speeds of around 100 miles per hour— in winter much faster. In fact, In February 2020 it helped a plane fly from New York to London in less than 5 hours. Naturally, such a strong flow of air can have profound effects on the weather. If the polar vortex (a fast spinning current of freezing air centered on each pole) is loose, the jet stream pulls the cold air out, leading to freezing, snowy winters. If itis tight, the jetstream misses it and the winter is mild and dry. Across the northern hemisphere, air south of the jet stream is warmer than air to the north, so that as the jetstream folds in on itself, like a loop in a river, it pulls weather sideways. That’s what happened with Hurricane Sandy in 2012, when a fold in the jetstream dragged the weather system from the Atlantic onshore in New Jersey. 8. The author's purpose in using “but” in the third line is primarily to: ‘A. draw attention to the speed of the jetstream. B, counter previous details about how fast the jetstream moves. C. warn readers the following information is contradictory. D. contradict the implications of the word “meander.” 9. Asit is used in line 5, the word naturally can best be replaced by which of the following words, with no change in meaning? A. physically 8, obviously C. genuinely D. geographically 10 10. The best evidence that Hurricane Sandy was dragged onshore by a fold in the jetstream is: ‘A. The jetstream runs roughly west to east. B, The jetstream moves much faster in winter. C. When the polar vortex is loose, cold air is dragged out. D. Air south of the jet stream is warmer than air to the north 11 ANSWERS Informational Text - Basic Quiz 2 ‘The passage says that: “at the time it was built, computer-aided design was in its infancy, and steel and concrete construction at the limits of technology,” suggesting the project was ambitious. (A) is incorrect because that’s what Jorn Utzon wanted— and the building is impressive—but the passage doesn’t tell us that. (B) is incorrect, because the design is unrelated to the nationality of its creator. (D) is incorrect because the author merely suggests it was challenging. What the author means by the structure shifting with every perspective is that no matter which angle you look the Opera House from, you get a different view—a new idea of the shape. By adding the word “economically” the author implies that casting the arcs in any other way (such as in individual molds for each size of strut) would have been too expensive. (B) is incorrect because while probably true, it has nothing to do with the author's use of the word. (C) is incorrect because the author using the word doesn’t, explain overspending. (0) is possible, but secondary—the fact that another method would have been expensive suggests the engineers were under financial pressure. If existing theories suggest modern man got to Europe after 60,000 years ago and the new evidence says they reached there 210,000 years ago, it suggests the theories need to be looked at again, and the speaker confirms this by saying: “We may have to come up with an entirely different picture of early modern man. (B) is incorrect because this particular Neanderthal skull was more recent than the H.sapiens one, but that doesn’t mean Homo sapiens arrived in Europe before Neanderthals. (C) is incorrect because while theories about early man may need re-evaluating, that doesn’t mean everything we know about early man is incorrect. (D) is incorrect too, because finding a very early example of a modern human in Europe doesn’t necessarily mean humans reached Europe first, just that datable evidence from elsewhere is scarce. The interviewer asks for clarification when he asks, “Older than 170,000 years?” and the scientist's "substantially older” confirms his previous statement that: “the real surprise was that the other skull was Homo sapiens—and it was older.” (A) is. incorrect because he is not exaggerating. (C) is incorrect because he is confirming what the interviewer has clarified. (D) is incorrect because hedging means sidestepping or equivocation 2 10, ‘The main idea of the passage is—contrary to all previous beliefs and despite the “Out of Africa Theory”—modern man reached Europe 210,000 year ago, and that could explain DNA evidence of Neanderthal’s interbreeding with H. sapiens much earlier than their arriving in Europe 50,000 years ago could explain. (A) is incorrect because the passage doesn’t claim there wasn't a mass migration, but that people left before it took place. (8) is incorrect because the passage doesn’t claim modern man evolved in Europe. (0) is tricky, because it is true, but does not summarize the main idea The article can point out the main idea as well as an interview for audiences that aren't experts—as evidenced by the previous question. (A), (B) and (C) are incorrect because those are all clear advantages of an interview over a written statement. The word “meander” not only means to move in a serpentine way, but also to stroll, amble, or move slowly, so inserting “but” before giving the speed, counters this meaning. (A) is incorrect because “but” suggests contrast, not emphasis. (8) is incorrect because no previous details are given. (C) is incorrect because the use of “put” doesn’t warn that the information about the jetstream going at least 100 miles an hour) is contradictory, but that the previous implication suggested by the word “meandering” was. Naturally can mean all of those things, but in this context it means clearly, logically, or obviously. The passage starts by explaining that the jetstream runs roughly west to east—which would mean the hurricane was over the land and got dragged towards the Atlantic— not the other way around. (8) is incorrect because this point is irrelevant to a folding jetstream moving contrary to the prevailing direction. (C) is incorrect because the polar vortex suggests air being pulled down from north to south, not from the Atlantic onshore, which is east to west. (D) is incorrect because this explains how a fold in the jetstream can occur, but still points to a north-south flow. 13 ©YBtong Reading Questions for the MAP Exam® 8" Grade Informational Text - Basic 10 Questions Quiz 3 Directions: Identify the choice that best completes the statement or answers the question. Read the following passage. In the fossil record there are birds and bird-ancestors that share characteristics with both modern reptiles and birds. They have feathers, but also teeth: modern birds don’t. Eggs are vulnerable, so not having to spend several months growing teeth before hatching increases the chance of survival. But what prompted the evolution of flight? There are a number of theories: escaping from predators, accessing food, rapid progress from place to place, or even freeing up the back legs for attack or defense. There are also debates as to whether flight began from the ground in a series of hopping and lifting motions (the cursorial theory], or from trees in a series of jumps or gliding motions (the arboreal theory). It now appears that the first preceded the second. A scientist, named Ken Dial, studied the way birds use their wings to run up hill. He filmed birds on ramps from just after they hatched until maturity, increasing the angle of inclination slightly day by day. He noticed that as chicks mature and their wings become stronger and their feathers longer, they are able to run up steeper and steeper slopes. Eventually adults can run up vertical surfaces—such as tree trunks. Probably a means of escaping predators (or catching food}, once up in the trees, the proto birds then learned how to get safely down again by spreading or flapping ‘their wings. 14 1, The purpose of the passage is primarily to: ‘A. provide an overview of the mechanics and key operations of wing function. 8, offer evidence that flight evolved as a means of escaping ground-based predators. C. compare the development of evolutionary strategies in both birds and lizards. D. analyze Ken Dial’s techniques for studying the motion of birds when faced with inclines. 2. In Ken Dial’s experiments, the ramps up which the chicks ran could best be described as: A. gradually inclined B. incrementally oriented. C. progressively steeper. D. increasingly higher. 3. Based on the passage, what may be inferred about the evolution of flight in bats? ‘A. Nothing: bats are mammals and share few characteristics with birds. B. Little: bats likely evolved flight around the same time as birds, but that’s all C. Something: bats probably evolved flight in response to predation in much the same way as birds. D. A great deal: lack of feathers and cumbersome movements point to the arboreal theory of flight. Read the following passage and grap! Passage The following passage is from an article on standardized testing. Standardized testing means every child in the U.S. education system is tested the same way. There are no easy exams for favorite students; no special tests for privileged schools. Many standardized tests are scored by machine, meaning that the scoring is free from bias. Moreover, it means that colleges and employers are able to choose the best and the brightest students based on their caliber, not on their address. A peer-reviewed analysis of testing between 1910 and 2010 suggests ‘that 93% of studies show that it has a positive effect on educational outcomes. One argument leveled at testers is that teachers stop providing an all-round education and teach to the test instead. However, the curriculum is carefully thought out by experts: if it wasn’t important, it wouldn’t be on the test, and that’s why teachers teach it. It focuses schools, parents, and students on essential skills and necessary content. Instead of teachers teaching their pet topics and their students losing out, they are now accountable, and—according the U.S. Department of Education—standardized tests eliminate “time wasting activities that don’t produce learning gains.” 15, Graphic: The following graphic is from a presentation on standardized testing. Results of PISA Test pre-and post- standardized testing 600 500 400 Average | US. Top | Average | US Top 2000 2018 Reading Math mScience lepeting countries: 28 OFCO and 4 non-OECD. 1 2028 there ware 79 participating counties: 35 OECD and 4 non-OECD Source: Progrem for International Student Assessment Cooperation and Development (OECD) SA), Organica 4, Based on the passage, the author's attitude toward standardized testing can best be described as one of: ‘A. approval for the impartial way in which the tests are administered and scored. 8, neutrality when comparing academic outcomes before and after adoption of standardized tests. C. appreciation for the progressive, unique opportunities they provide to teachers. D. concern that their implementation may leave some students at a disadvantage. 5. The claim that “colleges and employers are able to choose the best and the brightest students based on their caliber, not on their address” suggests that socio-economically: A. the tests are universally applied. B, the results are rigorously checked. C. the students are not identified, D. the schools are anonymous. 16 6. Based on the chart, which of the following statements would the speaker most likely make? A. Independent testing suggests that in 18 years, educational outcomes in the U.S. have fallen in real terms. 8B, The PISA results show that the U.S. is slipping in the rankings for reading, math, and science, despite 18 years of standardized testing. C. According to the OECD, standardized testing has improved outcomes in the majority of the nations tested, but not in the U. D. According to the chart, the U.S. is ahead of the average compared to 18 years ago, suggesting that standardized testing is working. 7. “A claim that 93% of studies show standardized tests improve academic performance was based on a handful of studies over 100 years, many unscientific and none independent.” Would the author of Passage 1 or the speaker presenting the chart most agree? ‘A. The author: the passage refers directly to the peer-reviewed analysis of testing between 1910 and 2010 and its positive results. 8, The author: the study showed improvement in academic outcomes, regardless of the fact, that methods a century ago may have been different. C. The speaker: any improvements in educational achievement over the past 100 years could be for a variety of co-factors. D. The speaker: the chart is about independent analysis of standardized testing and concentrates on the relevant 18 years. Read the following passage. ‘The hydrological cycle is familiar to most children by the time they reach middle school. Water evaporates, forms clouds, makes rain, and falls in the oceans for the process to start again. The rock cyde is less familiar, not because it is less visible, but because it takes place on vast time scale, There are different kinds of rock, including igneous and metamorphic, but sedimentary rocks weather most, easily. Trees, lichens, or other plants can break it inte small pieces, acid rain can eat it away, and wind and water can wear it dawn. Once particles have eroded, streams and rivers carry them downstream to lakes or the sea. Over time, they settle and compact under the weight of water, eventually turning into rock such as limestone and sandstone again. If the particles become buried underground because of tectonic plate action, the rock becomes compressed, but also heats up, changing into slate or marble, types of metamorphic rock. As plates continue to shift, a process called uplift can bring them to the surface again. Meanwhile, rock can become so hot it melts. Under pressure, itis extruded in the form of magma, forming igneous rocks such as basalt or granite. They take thousands of yearsto erode, but eventually, they toc crumble away. 7 8. According to the passage, a fundamental difference between the hydrological cycle and the rock cycle is A. The hydrological cycle is more familiar; it is usually taught in school. B, The hydrological cycle is a closed system; the water just changes form. C. The hydrological cycle is generally rapid; water can be recycled in days. D. The hydrological cycle is more evident; rain, fog, and snow are very visible. 9. The passage primarily focuses on: ‘A. how rocks are eroded, compressed, and cooked. B, the rock cycle as it pertains to sedimentary rocks. C. the different types of rock, including metamorphic. D. the essential difference between igneous and other rock types. 10. As it is used in this passage, “pressure” most nearly means: A. immense weight. B. severe stress, C. great strain. D. huge force. 18 ANSWERS Informational Text - Basic Quiz 3 The passage was written to provide evidence that that the cursorial theory is correct: birds evolved flight from the ground up, and likely started running up tree trunks because they were being chased by predators. They had to be ground-based because flight had not yet evolved. (A) is incorrect because the passage only explains how birds use their wings while running. (C) is incorrect because the passage says there are similarities between lizards and birds—but that's all. (0) is incorrect because the passage explains Dial’s experiments but does not attempt to analyze the techniques he used to study the birds. ‘The passage says that Dial was “increasing the angle of inclination slightly day by day,” so itis best to sum up the ramps as becoming “progressively steeper.” (A) is, incorrect because “gradually inclined” suggests they were not inclined to begin with. (8) is incorrect because the movements were incremental, but the orientation was not. (D) is incorrect because the slope of the ramp became steeper and steeper, not higher and higher. The passage doesn’t mention bats and we can infer nothing about the evolution of flight in bats based on this passage. (B) is incorrect—we can’t assume from the passage that flight evolved in bats and birds at the same time because the passage says nothing about it. In fact, bats evolved around 51 million years ago, while birds evolved 135 million years ago. (C) is incorrect because the passage gives us no clues. In fact, bats most likely evolved flight to find food rather than to escape predators, since their main predators are birds. (D) is incorrect because while itis 2 logical assumption, we have no clues in the passage that point towards this The author expresses approval for their impartiality saying “every child in the U.S. education system is tested the same way,” and that “scoring is free from bias.” (B) is incorrect because the author claims academic outcomes have improved. (C) is incorrect because the passage says, “Instead of teachers teaching their pet topics and their students losing out, they are now accountable,” rather than providing “unique opportunities.” (0) is incorrect because the passage says that “no easy exams for favorite students; no special tests for privileged schools” suggesting that no one is disadvantaged. 19 Rather than choosing students based on which state or town they live in, or which school they go to, favoring “wealthy” students or those from particular neighborhoods or with certain backgrounds, the same tests being administered everywhere means that in socio-economic terms, the students are not identified. (A), (8), and (0) are incorrect because they are not socio-economic factors. The chart shows that the top countries got better scores in 2018 after 18 years of testing than they did in 2000—but the U.S. didn’t. Its reading results were unchanged, its math results fell, and its science results were virtually unchanged. The gap between average scores and the U.S. may look encouraging, until you read the note at the bottom of the chart. The average score has likely fallen because so many more non-OECD countries—often those with fewer resources—took place in the 2018 tests. (A) is less likely, partly because although the Math results were slightly lower, the U.S. held its own in terms of reading and science. (C) is incorrect because the chart doesn’t tell us whether the “top” nations (far right bars of each group) are in the majority or not. (0) is incorrect because the speaker would most likely argue that standardized testing is NOT working, or that the evidence is—at best—inconclusive, since one score in the U.S. went down, and one stayed the same. Furthermore, the gap between the average OECD score and the U.S. has opened up, likely not because the U.S. has improved in relative terms, but because the average has dropped in relative terms, due to the inclusion of so many poorer, less advantaged non-OECD countries. ‘The speaker would likely agree with the statement because the chart only considers the 18 years since standardized testing began, and it focuses on independent testing, not on testing or studies done in the U.S. by the U.S. (A) is incorrect, because while the statement and the passage both refer to the same study, they draw opposite conclusions from it. This statement claims the studies were “unscientific” and none Were independent. (8) is incorrect because the statement is critical of the report saying it relied on only a handful of studies, and while methods may vary over time, the passage says they were “unscientific.” (C) is incorrect because while there may be co-factors, we learn that the study is about standardized testing, even though tests themselves may have been different in the past. ‘The passage says that the rock cycle: “takes place on vast time scale.” (A) is incorrect because while the hydrological cycle may be taught in school, so isthe rock cycle, and that isn’t a fundamental difference in the cycle. (8) is incorrect because although true, it is not mentioned in the passage. (0) is incorrect because the passage specifically says one of the main differences is “not because its less visible.” 20 10. The focus of the passage is the rock cycle when it comes to sedimentary rocks. It describes how they are weathered, break into pieces, get washed away, compacted, and then turned into rocks again. (A) is incorrect, because although the passage focuses on how rocks are eroded and compressed, it doesn’t mention rocks being “cooked”—they heat up and melt, but they aren’t cooked! (C) is incorrect because the passage mentions all three rock groups but doesn’t focus on their classifications or differences, (D) is. incorrect because the passage explains how igneous rocks are formed but doesn't stress the differences between igneous rocks and other types. The pressure referred to in this passage is “immense weight”: the weight of tons and tons of earth compressing the sediment below. (8) is incorrect, because that refers to emotional or psychological pressure. (C) and (D) are incorrect because they refer to power or stress (tensile, or stretching = strain), not to being squashed by immense weight. 2 ©YBtong Reading Questions for the MAP Exam® 8" Grade Informational Text - Proficient 10 Questions Quiz1 Directions: Identify the choice that best completes the statement or answers the question. Read the following passage. The idea that there might be life on Mars isn’t new. In 1996, scientists claimed that a piece of the red planet, which had arrived here as 2 meteorite, “pointed!” to life on Mars. The rock sample contained three possible clues. First, the rock contained magnetite, a crystal form of iron oxide which is the byproduct of certain bacteria. Second, the mix of minerals and carbon compounds was the same as that produced on Earth by certain microbes. Finally, under a Powerful microscope, the scientists could see segmented chain-like patterns similar to the shape of some Earth-bound bacteria. Unfortunately, all of this evidence could equally be explained by chemical and geological processes, and few scientists now believe the sample is proof of Martian life, Yet it did spark a debate and renew interest in Mars. In the past two decades, evidence has emerged that suggests that in the past, conditions there were much the same as on Earth—but something went wrong. Whereas life on Earth evolved, life on Mars may have begun, and then stopped. Why that happened is the current focus of NASA’s Mars research, and new rock samples are being collected that—in the future—could answer the life-on-Mars question once and for all. 2 1. The author puts the word “pointed” in quotation marks primarily to: AA. indicate that the evidence was limited, B. suggest the evidence was open to interpretation. C. highlight the scientists’ caution about the claim. D. underscore the doubtful nature of the evidence. 2. The best evidence that life could potentially have existed on Mars is: A. the existence of magnetite. 8, the particular mix of minerals and carbon compounds. C. the segmented chain-like patterns indicative of bacteria. D. the conditions being much like those on Earth. 3. As itis used in this passage, the word “certain” most nearly means: A. clear, 8. particular. C. evident, D. selected, Read the following two passages. Passage 1 ‘The US Mint produces twice as many pennies as nickels, dimes, and quarters combined. They are made of copper-plated zinc and cost 1.8 cents to make—almost twice as much as their face value. Each one represents a loss to the taxpayer. In a number of countries, pennies have already been withdrawn from circulation. The Department of Defense banned them on overseas military bases forty years ago because they were too expensive to ship out. Itis estimated that around two thirds of pennies, once issued, are never used again; many end up inthe trash. Furthermore, when they are used (more often by women than men, apparently) they slow down the speed of transactions, adding around 2 seconds to each cash purchase. Philip Diehl, former director of the U.S. Mint even claimed that “if you earn more than the minimum wage, you're losing money stopping and picking up a penny on the sidewalk.” 23 Passage 2 The penny has a place in people’s hearts. When asked if people wanted to ditch it or save it the majority (55%) wanted to keep it. Carefully hoarded in piggybanks, itis a symbol of thrift and much loved by charities. In 2009, the Leukemia and Lymphoma Society said that by collecting pennies— actual pennies—schoolchildren had collected $150 million dollars-worth for essential research and ‘treatment for cancer. Compared to nickels, as a percentage of their face value, they cost less to make, and they're often in circulation for 20 years. Without them, the Mint would have to make more S-cent pieces, which would cost them an additional $10 million dollars each year. Furthermore, it would impact children and families on low incomes, because shopkeepers would be forced to round prices to the nearest five cents, and the majority would be unlikely to round prices down. Typical pocket money wouldn't stretch as far, and in fact, when Canada abolished the penny, research showed that rounding up cost customers the equivalent of around $2.5 million dollars in the first year alone. 4, The central claim of Passage 1 is that pennies: ‘A. cost too much to make, 8B. are usually wasted or thrown away. C. waste time. D. weigh too much to transport. 5. Based on Passage 1, which of the following groups would most likely agree with the author that pennies should be withdrawn? A. soldiers B, seniors C. shopkeepers D. schoolchildren 6. Why does the author of Passage 2 include the phase “actual pennies”? ‘A. to highlight the work undertaken by the children B, to clarify the point as “pennies” is often used to mean money in general C. to emphasize the physical quality of the collections. D. to indicate the slow and painstaking nature of the charity collections 24 7. Which choice best sums up the tone of each passage? ‘A. Passage 1 is forthright, Passage 2 conciliatory. B, Passage 1 is assertive, Passage 2 resentiul. C. Passage 1 is negative, Passage 2 positive D. Passage 1 is pragmatic, Passage 2 emotional. 8. In terms of their audience, the passages differ in all of the following respects EXCEPT: ‘A. Passage 1 would appeal to employers, while Passage 2 would appeal to employees. B. Passage 1 would appeal to the government, while Passage 2 would appeal to charities. C. Passage 1 would appeal to men, while Passage 2 would appeal to children, D. Passage 1 would appeal to richer people, while Passage 2 would appeal to poorer ones. Read the following passage Ballads are narrative songs. Though the form differs, they are a European tradition which reached its peak in the Middle Ages, at a time when most communities had a purely oral tradition. Even now, in some communities where modern life is remote, folk singing continues as it always has. Russia, Spain, and England have impressive collections of ballads, and around 300 are shared stories common across the continent, forming part of the balladic tradition in, for example, Denmark, Greece, and Scotland. Most ballads involve limited characterization, with the action being the focus. Drama is achieved though question and answer, repetition or choruses, and the themes are universal: romance, heroism, or horror. One that has dozens of variations is Long Lankin, which relates how a sinister stranger waits until a lord rides away and then tricks his lady into opening the door before murdering her and her child. While the stories are often shared, the forms are not. In Britain and America, for example, ballads usually rhyme and are divided into verses; in Russia and the Balkans, they generally don’t. In Spain and much of Scandinavia, ballads Use assonance instead of rhyme, but in the former there are no verses, and in the latter, ballads are usually divided into quatrains. Whatever their form, ballads are part of the folkloric tradition and are undoubtedly here to stay. 25, 9. In which of the following traditions are ballads divided into verses? i. Britain ji, America iil, Russia iv, The Balkans v. Spain vi, Scandinavia A. vionly 8. iand ii only C.ii,iv, and v Dai, ji, and vi 10. The author’s explanation of the ballad of Long Lankin best illustrates which point? ‘A. “Russia, Spain, and England have impressive collections of ballads.” 8. “Most ballads involve limited characterization, with the action being the focus.” C. “The themes are universal: romance, heroism, or horror.” D. “While the stories are often shared, the forms are not.” 26 ANSWERS Informational Text - Proficient Quiz 1 Emphasizing the fact that the evidence only “pointed” to life on Mars suggests that the scientists were being very tentative about their conclusions. The evidence hinted at life on Mars, but was by no means definitive. (A) and (8) are true statements, but don’t explain the author's use of quotation marks around “pointed”; (D) is incorrect because the passage doesn’t suggest that at the time the evidence was doubtful: doubts came later. The passage says that “evidence has emerged that suggests that in the past, conditions there were much the same as on Earth—but something went wrong,” suggesting that if it hadn't, there would life on Mars even today. The rest of the evidence is what scienti 's thought they were seeing in the meteorite samples. The word certain can mean all of those things, but in the context of this sentence, it means particular: the mix of minerals and carbon compounds was the same as that produced on Earth by particular microbes The central claim or main argument in Passage 1 is that pennies are simply not, financially viable. They cost more to make than they are worth. Arguments (B), (C), and (0) are also correct, but they are supporting arguments. The passage says “they slow down the speed of transactions, adding around 2 seconds to each cash purchase,” but they would also add to the time it takes shopkeepers to count and bag the coins at the end of each day, ready to take to the bank. (A) is incorrect because active service military bases don’t use pennies anyways. (B) is incorrect because the passage doesn’t mention seniors at all. (D) is incorrect because although children are not mentioned in Passage 1, they would not earn “the minimum wage” and therefore any penny would be valuable to a child. The term pennies is often used as a synonym for money. For example, saying someone has to watch their pennies (be careful with money) or they are saving their pennies (saving their money). Often people don’t actually mean one cent coins, but in this passage the author is talking about just that—pennies that children collected one by one to donate to charity. By adding "—actual pennies—"the author is explaining that the term is to be taken literally. 7 10. In terms of tone, Passage 1 is all business; it is matter of fact and based firmly in the practicalities—pennies cost too much to make, they are wasted, too heavy to ship and slow commerce down—so the tone is pragmatic. In Passage 2, itis much more emotional, explaining that “The penny has a place in people's hearts,” that itis “a symbol of thrift” and “much loved” by charities. (A) is incorrect because although Passage 1 is forthright, Passage 2 is not conciliatory. (B) is incorrect because although Passage 1 is assertive, Passage 2 is not resentful, but passionate. (C) is incorrect because Passage 1 is against the use of the penny, but itis not negative in tone, nor is Passage 2 positive. The passages would appeal to different people. Passage 1 would appeal to the government (pennies cost 1.8 cents to make), employers (pennies slow down transactions), men (pennies are used more by women), and richer people (people who earn more than the minimum wage). Passage 2 would appeal to charities {pennies are often donated), children (they collect them, save them, and get them in pocket money), and poorer people (abolishing the penny would mean prices going up), but it would not necessarily appeal to employees. Employees are the ones who handle pennies in shops, have to count them to be banked and so on, and many would earn above the minimum wage or be men, so this choice is untrue. ‘The passage says that “In Britain and America, (iand ii) for example, ballads usually rhyme and are divided into verses” and “in the latter, [Scandinavia] (vi) ballads are usually divided into quatrains,” but in the other countries they are not. The ballad described comes under the category of “horror” (it tells of a sinister character who deliberately sets out to murder two people), soit best illustrates that “The themes are universal: romance, heroism, or horror.” (A) is incorrect, because the passage doesn’t say precisely where Long Lankin originated. (8) applies to what the author tells us about Long Lankin, but is not the best explanation of why the author includes it. (D) is incorrect because we don’t know if the many variations of Long Lankin take different forms. 28 ©YBtong Reading Questions for the MAP Exam® 8" Grade Informational Text - Proficient 10 Questions Quiz 2 Directions: Identify the choice that best completes the statement or answers the question. Read the following passage. Computers—as we know them—didn't exist in 1950. Nonetheless, a British mathematician called Allen Turning (sometimes called the father of computing) devised a test to try and determine whether a machine was intelligent. The test is simple enough. People ask questions of “someone” in another room by typing them on a keyboard. They see responses printed on a screen. They can ask anything they want. At the end of the session, they decide if their counterpart is a person or a ‘machine (in which case it passed the Turing Test). So far, no computer has, and that’s despite some amazing achievements in the field of artificial intelligence (Al). According to one view, that’s because the Turing Test is misrepresented. Turing didn’t design it to see whether a computer could pass itself off as human (the basis of many Scifi films) but whether it could think as intelligently as. a human. And that, of course, depends on how intelligent the questions are. 1. The author's use of quotation marks around “someone” indicates that at least some of the time, the person in question is: A. obscure. 8. plural . unknown, D. non-human 29 2. The author's view of the Turing test can best be characterized as: A. useful in limited circumstances. 8B. only as good as the questions in it. C. now outdated by more sophisticated techniques. D. of its time, and irrelevant in connection with Al. 3. The primary purpose of the passage is to: ‘A. describe the purpose and scope of the Turing Test, 8. explain that the Turing Test is often misrepresented, C. discuss the origin and development of Al testing, D. determine the effectiveness of the Turing Test. Read the following interview and passage. Passage 1 Interviewer: Dr. Simons: Interviewer: Dr. Simons: I'm joined by Dr. Simons of the University of Minnesota, who is calling for middle and high schools in the U.S. to start at 9:00 or 10:00 a.m. That's right. I'm a parent, and it’s a real concern. By the time kids get to high school, 87% of them are sleep deprived. They tend to stay up two hours later than they did when they were younger, but they are still getting up for an 8:00 a.m. school start time. We've just completed some research on this. It took three years and while it covered just three states, we think the results are indicative We found that just shifting the start time by half an hour, to 8:30 a.m., kids had noticeably better test scores, and schools saw a massive reduction in teenage car accidents—down 70%. And that simply has to do with not enough sleep? Yes. It means kids get up late, sometimes miss early classes, come to school without breakfast... ey're drowsy and slow reacting behind the wheel and tired and lackluster in class. If shifting the school day by 30 minutes stops all that, then we have a duty to do it. 30 Passage 2: ‘Teenagers undergo a shift in their internal body clocks which coincides with puberty. According to ‘The Parents’ Union its related to melatonin production and circadian rhythm, and is a biological change. Left to their own devices, therefore, teenagers will stay up till midnight and emerge around 9:00 a.m. in the morning. By 10:00 a.m., they are awake and raring to go. That pattern doesn't fit with the typical school day, since most establishments start at 8:00 a.m. The American Academy of Pediatrics, urging for a change in school start times, stated that, “Insufficient sleep represents one of the most common, important, and potentially remediable health risks in children, particularly in the adolescent population.” And, according to the University of Michigan it leads to “lower academic achievement ... higher rates of absenteeism and tardiness, and decreased readiness to learn.” Already 1,000 U.S. schools in seventy districts have opted for an 8:30 a.m. start time, but Oxford University, England, thinks that 10:00 a.m, would be even better. ‘They are currently doing randomized trials to find the optimum start time for sleepy teens on both sides of the pond 4. During the interview, what does Dr Simons imply about sleep deprivation? A. Itis entirely preventable if parents were stricter about bedtimes. B, It becomes worse and more widespread as children go up through the grades. C. Schools should do more to make allowances for sleep deprived teenagers. D. Car accidents would stop and test results would be perfect if a 10:00 a.m. start time were adopted. 5. Passage 2 suggests that the mismatch between a teenager's wake-sleep cycle and the organization of the school day is: ‘A. physiological in origin and should be accommodated with staggered school hours. B. a perfectly natural part of the shift toward adulthood and can be medically controlled. C. now well documented and it behooves governments to mandate starting half an hour later. D. easily remedied, but requires data from randomized trials in England to be accurate. 6. Compared to the interview, the article has an advantage, in that: A. it incorporates several different points of view. 8, the information it gives can be verified C. it introduces evidence from multiple expert sources. D. a written article does not convey the writer’s opinion. 31 7. Which choice best states the relationship between the two passages? ‘A. Passage 2 provides scientific refutation for the perspective offered in Passage 1. B, Passage 2 illustrates the practical difficulties of a proposal made in Passage 1. C. Passage 2 takes issue with the primary argument of Passage 1. D. Passage 2 offers an explanation for a phenomenon mentioned in Passage 1. Read the following passage Aside from competition and predation, there are three types of symbiotic relationships in nature. In parasitism, the organism, (such as a flea or mistletoe) benefits, while the host (a dog or a tree) suffers. In commensalism, while the organism derives some benefit, the host is unaffected: it neither benefits nor suffers. In mutualism, both parties get something out of the relationship. Pseudomyrmex ants, for example, live in acacia trees. The trees contain sugar- bearing veins which the ants tap for food, and clusters of thorns which provide shelter. In return, the bijou bodyguards defend the tree against herbivores. They exude a venom which is toxic (and painful), thus discouraging browsing on the leaves. They also, according to new research by the Max Planck Institute for Chemical Ecology, provide protection against microbial pathogens—plant diseases caused by bad bacteria. Apparently they carry good bacteria on their legs; as these rub against the acacia’s leaves, they destroy the bad bacteria and help keep the plant in good shape. 8. Based on the passage, a barnacle which attaches itself to shark, taking advantage of scraps of food left from the shark’s prey, provides a clear example of which type of symbiotic relationship? ‘A. Competition, as the barnacle is competing for food from the same source as the shark. B, Parasitism, as the barnacle uses the shark as a host and gives nothing in return, C. Commensalism, as the shark is indifferent to the barnacle and derives no harm from it. , Mutualism, because the barnacle cleans up after the shark and feeds on its leftovers. 9. What is the effect of the expression bijou bodyguards on the passage? A. Itmakes a serious scientific passage more accessible to a general audience. B, It introduces a light-hearted tone to what would otherwise be a dry, academic text. C. It suggests the author is amused at the thought of something so small protecting something so large. D. It adds a touch of mockery to the passage, suggesting that symbiotic relationships can exist in unlikely places. 32 10. Compared to ants, that tap a vein to gain sustenance, herbivores graze on acacia leaves devoid of ant protection and are portrayed as: ‘A. cunning, B, leisurely. C. haphazard, D. unfortunate 33, ANSWERS Informational Text - Proficient Quiz 2 The passage says: “People ask questions of “someone” in another room,” and later that “they decide if their counterpart is a person or a machine,” so the “someone” is sometimes a machine (a computer). (A) is incorrect because the “someone” is obscured (meaning hidden) rather than obscure (mysterious). (B) is incorrect because the passage doesn’t suggest a single person could be asking questions of multiple intelligences. (C) is incorrect because while the person in the other room may well be unknown to the questioner, that isn’t why the author put quotation marks around “someone.” Rather, it indicates that the author could have written “someone or something.” The author concludes the passage with his/her opinion, introduced by “of course,” which is that whether computers can think as intelligently as a human “depends on how intelligent the questions are.” (A) is incorrect because the author limits the test’s usefulness purely to the quality of the questions. (C) is incorrect as no more sophisticated tests are mentioned. (0) is incorrect because the test was devised before there were computers. The purpose of the passage is to describe the purpose of the test (what itis designed to do) and its scope (the range of what it can do). (B) is incorrect because the author writes: “According to one view, that’s because the Turing Test is misrepresented.” (C)is incorrect because the development of Al testing is not discussed. (0) is incorrect because the focus of the passage is not on how good the test is when it is used for its proper purpose—we only know that a computer hasn’t passed yet—but to describe what itis supposed to do and how it works. Dr Simons says that: “By the time kids get to high school, 87% of them are sleep- deprived,” which suggests that a significant number are sleep deprived before they reach high school, but things just get worse as kids get older. (A) is incorrect because the passage doesn’t say anything about parents being stricter: Dr Simons just says he's a parent and he’s concerned. (C) is incorrect because Dr Simons doesn’t say schools should make more allowances—such as being lenient about tardiness or poor grades—but that they should alter start times. (0) is incorrect because Dr Simons doesn’t suggest perfection would result, just that car accidents would be reduced and grades would improve. 34 The passage says that the shift in teenagers’ timing is related to “melatonin production and circadian rhythm and is a biological change.” That means itis, physiological and beyond a teenager's ability to control—so schools need to offer staggered start times because they can change—teenagers can’t. (B) is incorrect, because while perfectly natural, the passage doesn’t suggest controlling it with medication. (C) is incorrect because although the passage indicates there isa lot of scientific evidence available, it doesn't say government should force schools to start 30 minutes later—rather it suggests research is ongoing and that two hours later might be better. (D) is incorrect because while school start times could be changed, it would not necessarily be “easy” (a family could have child in Grade 3 starting at 8.00 ‘a.m,, one in Grade 5 starting at 9.00 a.m., and one in Grade & starting at 10.00 a.m.) and is not dependent on data from England. The advantage of the article is that it can gather evidence from multiple sources, whereas the interviewer is talking to one expert from one university—a single source. (A) is incorrect because the article has a single point of view (the author's) but explains what other people say about the issue (multiple sources). (B) is incorrect because both spoken and written information can be verified—the only difference is when. (D) is incorrect because a written article can convey the author's opinion as well as the spoken word. Passage 2 differs from Passage 1 in that in the interview the problem is stated {teenagers wake up late and it leads to accidents and academic underachievement), The article restates that, but also explains why. (A) is incorrect because the science in Passage 2 doesn’t refute the perspective in Passage 1—it explains and confirms it. (B)is incorrect because the proposal in Passage 1 is to start schools 30 minutes later, and Passage 2 suggests longer than that, but points to the practical difficulties of getting teenagers to change their behavior—not changing start times. (C) is incorrect because Passage 2 doesn’t take issue with the primary argument—delay school starting time for teenagers—it just suggests half an hour may not be enough The passage explains that in commensalism, the host doesn’t suffer, but the organism benefits. Since the shark is “indifferent to the barnacle and derives no harm from it” but the bamnacle gets a surface to attach to and can mop up leftovers from the shark's dinner, then it is a commensal relationship. (A) is incorrect because the barnacle isn’t competing for food with the shark but takes advantage from the fact that sharks are messy eaters. (B) is incorrect, because only in mutualism is something given in return; in parasitism, the barnacle would have to take something from the shark or harm it in some way, and it doesn’t. (D) is incorrect, because the shark doesn’t benefit from the barnacle consuming microscopic pieces of fish or blood in the seawater surrounding them both after the shark has been feeding 35, 10. The passage isa dry, fairly academic scientific text, which the author lightens by referring to the ants as bijou bodyguards. (A) is incorrect because it doesn’t open up the text to the general reader: that would require use on non-scientific terms, simpler language, and so on. (C) is incorrect because although the author might well be amused, the question is what effect does the phrase have on the passage ~ not the author. (O) is incorrect because the phrase is more cute than contemptuous, so it isn’t mocking or derisory. By describing the herbivores as “browsing”,” the passage suggests that they are more haphazard in their search for food. The ants go straight to a vein and tap the sugar— the grazing animals wander from place to place looking for food, sometimes unsuccessfully. (A) is incorrect because “browsing” doesn’t suggest cunning. (B) is incorrect because browsing in terms of animals feeding means grazing, but itis easy to be misled by the leisurely connotations of the verb browse which also means to look through casually. (0) is incorrect because only those plants which have ants would mean the animals were unfortunate (because they’d be stung) 36 ©YBtong Reading Questions for the MAP Exam® 8" Grade Informational Text - Proficient 10 Questions Quiz 3 Directions: Identify the choice that best completes the statement or answers the question. Read the following passage. Put simply, stress is the body’s way of dealing with pressure from a particular event or situation. It triggers the same chemicals as the fight-flight response—a fundamental safety mechanism to help Us respond to danger. So if it a natural reaction, why is stress seen as one of the modern world’s biggest killers? According to Daniela Kaufer, a neuroscientist at the University of California, Berkeley, “Your stress response is crucial to your survival. it elevates your performance, is super- important for alertness and prepares you to adapt to the next thing that comes along.” This kind of stress is called eustress (eu- is simply a Greek prefix meaning “good”), and it’s the type that makes you do well on exams, smash it on stage, or make that winning strike. It stimulates the growth of new neurons, so stress helps people learn from things and know what to do next time. Stress becomes negative when that learning fails to take place, or when we have a stress overload because chemicals such as adrenaline are released but have no physical release—such as running away—to compensate. Itis that kind of stress than leads to strokes and heart attacks. 1. Which choice best supports Daniela Kaufer’s view of stress? A. “Stress is the body’s way of dealing with pressure.” 8B, “It triggers the same chemicals as the fight-flight response.” C. “it stimulates the growth of new neurons.” D. “Stress helps people learn from things and know what to do next time.” 7 2. Which choice best represents the different meanings of “simply” as used in this passage? ‘A. modestly; only 8. purely; basically C. easily; clearly D. plainly; just 3. According to the passage, stress is only harmful if: ‘A. new neurons fail to grow to accommodate stress. B. excess chemicals fail to lead to physical reaction, C. eustress fails to offer appropriate compensations. D. people fail to recognize it as a natural phenomenon. Read the following passage and chart. Passage: If the effects of global warming are to be mitigated, itis essential that energy costs are reduced. ‘The 2012 International Energy Conservation Code mentions several ways of doing this, including one that office workers and homeowners rarely think about: insulation. "If all U.S. homes were fitted with insulation based on the Code, residential electricity use would drop,” says Dr. Jonathan Lewy, professor of environmental health at Boston University. "People don't see insulation, so they don't think about it," says the CEO of The North American insulation Manufacturers Association (NAIMA). Yet a layer of material between the outer and inner walls of a building, under its roof covering, or even on doors and windows, can dramatically reduce the amount spent on heating and cooling. Costs can fall by up to 15%. As a result, there has been a surge in work for insulation installers as municipalities, business owners, and householders upgrade existing properties and construct new ones. NAIMA expects the job market to increase by around 30%, and that doesn’t include jobs in manufacturing insulation. As the drive towards other forms of insulation continues, those are set to rise too. Instead of insulation made from oil products (such as foam), consumers want non-toxic insulation from renewable sources that is easy to dispose of when a building comes to the end of its useful life, 38 Chart: ‘The following chart was shown at a presentation on Insulation, % Increase in Sales of Insulation in the U.S. 2016-2019 2009-2018 N. EE Aetectve/Reclort OB roeess _ aE Bion EE be ? NE gore NN fockwoot B cicesesoosuct £5 bee rom cred meters Mace trom renewable sources IN tae tom natural source tone, eta) Source: Freedonia Group insulation Report 2019 4. Which proverb best captures the view of NAIMA’s CEO when it comes to insulation? ‘A. There are none so blind as those that will not see. B, Prevention is better than cure. C. Achain is only as strong as its weakest link, D. Out of sight is out of mind. 5. In the statement “Costs can fall by up to 15%,” costs refer to: A. energy costs B. insulation costs C. building costs, D. installation costs 39 6. According to the chart, the greatest percentage increase in installation has been in: A. oil-based insulation. B. insulation from recycled materials C. insulation from natural products D. insulation from renewable sources. 7. Based on the chart, would the speaker at the presentation agree or disagree with the claims about manufacturing insulation made by the author of the passage? ‘A. The speaker would likely disagree because the chart shows insulation made from recycled materials, not which insulation can be recycled. 8B. The speaker would likely partly disagree as the chart makes no reference to non-toxic. insulation. C. The speaker would likely generally agree as the growth in non-oil based insulation, combined, far outweighs that for foam insulation. D. The speaker would wholeheartedly agree as the chart shows natural, renewable, and recycled insulation in recent years where there was none before. Read the following passage. According to many, sitting one high-stakes exam at the end of a period of schooling is unfair. There are many arguments in favor of not deciding someone's future on the basis of a single test, but not al carry weight. One claim is that the corticostriatal network of the teenage brain is not fully developed, which apparently means that teenagers simply can't judge how important certain workis to their final grade, The corticostriatal network is the part of the brain responsible for evaluating risk and reward. In one experiment, people of various ages were asked to play a video game, where the stakes were low: 20 cents for a correct move and -10 cents for a wrong one. They then upped the stakes: $1 for a correct move and -50 cents for a wrong one. The older the participant, the better they performed, adapting their moves when the stakes were high to minimize losses. Teenage participants lost significantly because they were unable to distinguish between risks. When it comes to those vital exams, therefore, the researchers claim that this proves that teenagers don't always understand how high the stakes are—hence the call for smaller, more frequent tests, and continuous assessment. 40 8, It can most reasonably be inferred from the passage that the author's attitude toward claims that teenagers don’t understand how important their studies are is one of: A. definite sympathy. 8, confused fluctuation C. slight skepticism. D. outright disbelief. 9. According to the passage, risk taking strategies corresponde with: AQ, 8, brain size. C. maturity. D. motor-skills 10. As itis used in this passage, the word “significantly” could be replaced by all of the following EXCEPT: A. dramatically. 8B. considerably. C. agreat deal. D. meaningfully. a ANSWERS Informational Text - Proficient Quiz 3 Kaufer's view is that “Your stress response is crucial to your survival,” so the author's statement that “it [stress] triggers the same chemicals as the flight-fight response” directly supports this view. It evolved to protect us from danger and help us “survive”. The other choices (A), (B), and (D) are all partly correct, but do not address the fact that stress helps you survive. In “Put simply, stress is the body's way of dealing with pressure,” means put clearly or stated plainly. In “eu-is simply a Greek prefix meaning ‘good’,” it means just or only, making D the only correct pair. (A) is incorrect because “modestly” is not a suitable substitute for the first use. (B) is incorrect because the two words are the wrong way round: purely is similar in meaning to the second use and basically similar to the first. (C) is incorrect because clearly means the same as just in its first use, but easily doesn’t match either use The passage states that: “Stress becomes negative when... chemicals such as adrenaline are released but have no physical release.” (A) is incorrect because the passage says that stress “stimulates the growth of new neurons, so stress helps people learn from things” not that neurons accommodate stress. (C) is incorrect because the passage doesn’t suggest that stress and eustress need to reach equilibrium. (D) is incorrect because although the passage clearly implies stress is perfectly natural, it doesn’t suggest that falling to recognize that fact leads to harm. The CEO of The North American Insulation Manufacturers Association (NAIMA) said “People don't see insulation, so they don't think about it," and this is consistent with insulation being “out of sight, out of mind.” (A) is incorrect, because that would imply that people see insulation, but fail to do anything about it. (B) is incorrect because while insulating a building would prevent energy loss, that isn’t what the CEO mentioned. (C) is incorrect, because although this might apply to the efficiency of a building with insulation being the weakest link iit is the wrong type, or non existent, or badly fitted, this isn’t what the CEO comments upon. The passage says it: “can dramatically reduce the amount spent on heating and cooling. Costs can fall by up to 15%.” That means the costs are those for heating and cooling or energy costs. (8), (C) and (D) are all plausible, but incorrect given the context, 42 10, c Natural products (stone, glass) are used in reflective and radiant insulation (an increase of over 4%) and in Rockwoo! (an increase of around 2%) and well as in some “Other” products, making the increase in excess of 6%. (A) is incorrect, because the difference between the grey and the black bar is just over 4%. (8) is a close second, because fiberglass has increased more than any other type (by over 5%) and itis, made of recycled materials. Another big increase is in “Other” materials, which also has a recycled symbol. (D) is incorrect because insulation from renewable sources in the form of cellulose increased by just over 3% and an unknown amount in “other” types of insulation. The author of the passage makes this claim about the manufacturing of insulation: “As the drive towards other forms of insulation continues, those [jobs] are set to rise too.” The chart also shows a huge increase in “other” types of insulation—from renewable and natural sources or made from recycled materials (and therefore usually recyclable), so the speaker would most certainly agree that jobs in that sector are likely to rise. (A), (B) and (C) are incorrect for the same reason: none address the ‘main claim that jobs will rise for “other” forms of insulation The author of the passage is highly skeptical or disbelieving of the claim that teenagers don’t understand how important their studies are. He or she says that some arguments against a single high-stakes exam may be valid but “not all carry weight.” He or she goes on to say, “One claim is that the corticostriatal network of the teenage brain is not fully developed, which apparentiy means that teenagers simply can’t judge how important certain work is to their final grade.” Finally, the author reveals doubts as to whether an experiment about gambling on the outcome of a video game is, proof that teenagers don’t understand how important their studies are by saying “the researchers claim that this proves...” (A), (B) and (C) are incorrect for the same reasons. The passage says, “the older the participant, the better they performed,” suggesting that risk-taking strategies are corresponde with maturity. (A) and (B) are incorrect because the passage doesn’t mention IQ or brain size—just that part of the brain is involved. (D) is incorrect, because while the experiment involved motor skills in playing the video game, that wasn’t what it was testing. The passage says: "Teenage participants lost significantly because they were unable to distinguish between risks.” That means they lost a lot, meaning (A), (B) and (C) are all possible substitutes, whereas (0) is not. Meaningfully means in a worthwhile manner, and can’t substitute for significantly in this sentence. 43 ©YBtong Reading Questions for the MAP Exam® 8" Grade Informational Text - Advanced 10 Questions Quiz1 Directions: Identify the choice that best completes the statement or answers the question. Read the following passage. ‘The United States is the world’s largest producer of geothermal energy and is home to the biggest geothermal field—the “Geysers” in California—housing twenty-two power plants spread over more than one hundred square kilometers. Geothermal plants don’t cause the same kind of pollution as those burning fossil fuels or raise the same concerns as those that rely on nuclear fuels. As a result, geothermal energy is bruited about as a clean, environmentally-friendly, and essentially “green” form of energy. Yet it gradually leeches the Earth's body heat and is not without problems of its own. Geothermal plants can visually impact the landscape with pipelines and exchange towers and cause noise pollution just as wind turbines or hydroelectric stations can. They also allow the release of carbon dioxide, which, while not a pollutant, is a greenhouse gas. In addition, they disturb land at driling sites. Most significant, however, is the saline or highly corrosive wastewater which emerges—still steaming—from a geothermal plant and must be safely disposed of. 1. The primary purpose of this passage is to: A. suggest that geothermal energy has been generally mischaracterized, 8. highlight the importance of geothermal energy plants in the United States, C. clarify the status of geothermal plants in environmental terms. D. emphasize the drawbacks of geothermal energy in terms of pollutants. a4 2. What evidence does the author present to suggest that geothermal energy is not sustainable as a source of energy? A. It disturbs land at drilling sites. B, It leeches the Earth’s body heat. C. Itimpacts the landscape. D. It causes noise pollution 3. What is the effect on the tone of the passage of the author's use of the word “bruited”? ‘A. Itadds to the suggestion that geothermal energy is both popular and commonplace, B, It emphasizes the commonly held beliefs people have of geothermal energy. C. It suggests the author's skepticism of geothermal energy’s credentials. D. It creates an informal, confidential tone which engages the reader. Read the following two passages. Passage 1 In recent years Paleo diets have become fashionable. They are based on foods that would have been available to our ancestors during the Paleolithic period (2.6 million years ago to about 12,000 years ago). They would have had meat and fish they could hunt, and fruit, nuts, or grains they could collect, but no beans or modern grains, and they would not have drunk milk or made yogurt or cheese, because they didn’t herd animals. Fats and proteins came from meat. Modern Paleo-diets are practically 50% meat, but in the past, in reality, hunters rarely had access to meat and in winter, many starved. Few people lived to the age of 30—more than half died before they reached 15. Certainly, when food was plentiful, the diet was healthy enough—rich in vitamins and fiber—but getting enough calories would have been difficult. According to paleoanthropologist Alison Brooks of George Washington University, the paleodiet was extremely varied, and ranged from the exceedingly rare 99% meat diet of the Yakut (an Inuit tribe in Alaska who live on seal meat and fish), to the almost 99% vegetarian diet of the Tsimane Indians in the Amazon (who live on tubers and fruit). Like these modern people, Stone Age peoples “ate what was available,” she explains, “and adapted accordingly.” 45 Passage 2 Supporters of the Paleo-diet—eating the same foods our ancestors would have eaten 10-12,000 years ago—claim that itis healthier and that our bodies are not evolved to eat a modern diet. One example they point to is milk. Some people are lactose intolerant, meaning they can’t process milk or milk products and until farming began and people began to keep goats and cows, milk simply wasn’t available, All mammal babies produce an enzyme called lactase which allows them to process their mother’s milk, but production in some populations stops at around 2 years of age. Researchers at University College, London, however, have traced a gene that ensures lactase Persistence into adulthood—meaning that people can digest milk, butter, and cheese as easily as, ‘a baby. The mutation occurred 5-6,000 years ago in response to the availability of milk, but only occurred in certain populations. It is common amongst those with European ancestry, for example, but rare among those of African ancestry. We have undergone other changes too—developing smaller teeth and jaws, for example. “The notion that we stopped evolving in the Paleolithic period simply isn’t true,” says geneticist Sarah Tishkoff of the University of Pennsyivania. "We're still evolving, so going back to a Paleolithic diet for most people simply isn’t feasible.” 4, A clear implication in Passage 1 is that meat protein: ‘A. was available to only a few groups during the Paleolithic meaning that modern Paleo-diets do not reflect what our ancestors really ate. 8. is more heavily consumed in modern Paleo-diets than it would have been in the Stone Age. C. varies significantly from place to place and over time, allowing few conclusions to be drawn ‘as to what a true Paleo-diet looks like. D. is less important to a healthy diet than protein that is derived from non-meat sources such as tubers and fruit. 5, Passage 1 suggests that few people in the Paleolithic period lived any length of time, most likely because: ‘A. longevity relies not only on a balanced diet but on medical care which would have been unavailable in the Paleolithic. B, their diet focused too much on meat and not enough on the fruit and vegetables necessary for vitamins and fiber. C. life expectancy in the Stone Age was limited by the poor living conditions and necessity to hunt large and dangerous animals. . fruit and vegetables were unavailable for several months of the year and hunting was difficult, so many people had insufficient food to see them through winter. 46 6. The author of Passage 2 claims all of the following about lactose intolerance EXCEPT: A. it common in all adult mammals. B, Itis a mutation dating back thousands of years. C. itis the main reason why people choose a Paleo-diet. D. itis sometimes overcome by lactase persistence. 7. Passage 2 most strongly suggests that the gene for lactase persistence: ‘A. may be one of several adaptations in response to changing diet. B. isa recent mutation which is still being passed on. C. evolved in response to alternative farming methods, D. is linked to populations relying on dairy produce for protein 8. What is the relationship between the two passages? A. Passage 2 challenges the main argument of Passage 1. B, Passage 2 reaches similar conclusions for different reasons to Passage 1. C. Passage 2 elaborates upon an idea introduced by Passage 1. D. Passage 2 draws different conclusions for the same reasons as Passage 1. Read the following passage. Legends differ from stories and myths in one important detail. [1] Somewhere, under all the twists and turns of plot, the characters, and the drama, there is a grain of truth. [2] But how far back do the stories go? In many cultures, storytelling was an oral tradition, with very few tales coming down to us nowadays from written sources. [3] However, researchers in Australia looking at traditional Aboriginal stories have been able tote the events in the tales to real, geological events Even now the stories are not written down but passed on from generation to generation—some secret from outsiders to this day. [4] Others have been rendered into English by researchers, and then correlated across the country, so that stories from one tribe are cross referenced to similar stories from different tribes in different parts of the country. Plotted on a geological timeline, they coincide to events from the last Ice Age when sea levels rose significantly and rapidly, and suggests that eye-witness accounts have been enshrined in an oral tradition which has remained unbroken for a staggering 12,500 years a7 9. The following sentence has been omitted. Where should it go? One exception is the epic of Gilgamesh which was written on clay tablets and dates from 3000 BCE A. Point 1 B, Point 2 C. Point 3 D. Point 4 110. What is the author's attitude to the claim that Aboriginal stories can be linked to real geological events? A. It proves that legends have a grain of truth in them, B, It is astonishing that real facts could survive being passed down for that length of time. C. Itis interesting that oral traditions survive even after the invention of writing, D. It suggests that investigating other cultures’ traditional stories could prove worthwhile. 48 ANSWERS Informational Text - Advanced Quiz 1 The author's main purpose is to set the record straight by saying that although geothermal energy is seen as “a clean, environmentally, and essentially “green” form of energy,” whereas it can cause pollution, issues with waste water, COz emissions, and damage to the landscape, and is in fact, not even a renewable source. In other words, it has been mischaracterized as “green.” (B) is incorrect as this is a supporting detail. (C) is incorrect as the author goes further than clarifying geothermal energy’s status: it claims it has been misrepresented. (D) is incorrect because pollutants are only one aspect mentioned in the passage. The author explains that geothermal energy steals or “leeches” heat from the core of the Earth (the Earth's “body heat”) so (although over a long time) it will eventually cause the Earth to cool, and in that sense is not renewable or sustainable. (A), (C), and (D) are effects of using geothermal energy but are not relevant to its sustainability in terms of being renewable—only in terms of being unpleasant. The verb to bruit (often used with “about”) means to gossip or spread rumor, and suggests that what is being said is untrue, so by saying that “geothermal energy is bruited about as a clean, environmentally-friendly, and essentially “green” form of energy” the author is showing his or her skepticism of the credentials of “green” and “clean” as given to geothermal energy. (A) and (B) are incorrect because they have no bearing on tone. (0) is incorrect because although it adds a touch of informality, it is not designed to take the reader into the author's confidence. Passage 1 says: “Paleo-diets are practically 50% meat, but in reality, hunters rarely had access to meat” and that that a high meat diet, like that eaten by the Yakut, was “exceedingly rare.” That means that someone eating a modern Palo diet would consume meat far more often than a typical person from the Paleolithic. (A) is incorrect because the passage says meat in general was scarce, but not that it was, available only to a few groups. (C)is incorrect because while itis true that diet varies over time and with place, that doesn’t mean experts don’t know what a true Paleo- diet would have been. (0) is incorrect because although the passage mentions that the Tsimane Indians in the Amazon live on tubers and fruit, it doesn’t say that this is a more important as a source of protein. 49 Passage 1 says that “in winter, many starved” and that “few people lived to the age of 30,” and suggests reasons for that. It says that the diet “was healthy enough” when food was plentiful—but it would not have been plentiful in winter (or early spring) before nuts, seeds, berries and tubers had grown. In addition, the passage says “They would have had meat and fish they could hunt” but that “in reality, hunters rarely had access to meat” suggesting that hunting food was very difficult. Combined, that means that many Stone Age people would not have been able to get through the winter. (A) and (C) are incorrect because although true, medical care and living conditions are not mentioned by the author. (8) is incorrect because the passage doesn’t say Paleolithic people relied too heavily on meat; rather, it suggests ‘modern so-called Palo-diets are too rich in meat. The author explains that lactase persistence—the ability to continue processing lactose in milk beyond baby years—is the mutation that occurred 5-6,000 years ago, not that lactose intolerance is the mutation. In all mammals, after babies are weaned, they lose the ability to digest lactose easily, so those people that can process it (albeit whole populations) are the ones who possess the mutated gene. (A), (C) and (D) are all true according to the passage. The passage says that the lactase persistence mutation occurred only in certain populations, such as in Europeans, but there would be no point in evolving the ability to process lactose if lactose wasn't an important part of your diet, and lactose comes from milk, so the implication is that the genetic mutation is linked tc populations reliant on milk protein. (A) is incorrect because the passage suggests there have been other adaptations (a smaller jaw, for example) but not that they are necessarily linked to diet. (B) is true, but is not indicated by the passage. (C) is incorrect because itis not farming methods that promoted the genetic change, but the reliance on dairy products. Passage 1 draws the conclusion that modern Paleo-diets are based on a false premise, because Stone Age people didn’t have access to as much meat as modern diets suggest, while Passage 2 draws the conclusion that modern Paleo-diets are based on a false premise because humans (at least some) have evolved to drink milk. Both passages are therefore dismissive of the modern Palo-diet but for different reasons. (A) is incorrect, because the main argument in Passage 1 is that a modern Paleo-diet does not reflect what our ancestors really ate, and Passage 2 does not refute that. (C)is incorrect because Passage 2 lights on a different aspect of the Paleo diet, rather than elaborating on something mentioned in Passage 1. (0) is incorrect because both passages reach similar conclusions but for totally different reasons. 50 10. ‘The missing sentence refers to an exception because it was written down, so logically it belongs after mention of written sources at point 3, (A) is incorrect in that position it would imply that Gilgamesh is an exception because it doesn’t differ from a story or myth. (B) is incorrect because in that position it suggests that there is no grain or truth in Gilgamesh, but mention of Gilgamesh is not about truth, but about being written down. . (Gilgamesh is an epic story in poem form from ancient Mesopotamia and contains references to Ice Age inundations of “flood” stories.) (D) is incorrect because it places Gilgamesh amongst Aboriginal tales and implies itis a story that is one of the few not “kept secret”, The author's attitude is one of astonishment that stories that were told of real events—the rising waters of an Ice Age inundation—could have been passed on by word of mouth, generation after generation for 12,500 years. The passage refers to a “staggering” 12,500 years. Oral traditions rarely survive 10 generations: the Aboriginal tales -verified by cross referencing completely different stories amongst discrete tribes in different places—have been passed down for more than 400 generations. (A) is incorrect because the passage starts with this premise. (C) is incorrect because the author doesn’t investigate the written tradition. (D) is incorrect because the author doesn’t suggest looking at similar findings amongst other cultures. 51 ©YBtong Reading Questions for the MAP Exam® 8" Grade Informational Text - Advanced 10 Questions Quiz 2 Directions: Identify the choice that best completes the statement or answers the question. Read the following passage. According to traditional textbooks Mesopotamia was the earliest civilization. It began in 3,500 BCE along the Tigris-Euphrates river system, from (on today’s maps) south eastern Turkey to the Persian Gulf. No-one would dispute its being a civilization, but therein lies a problem. What is a Civilization? According to most sources, it needs to have some cities—it can’t be a nomadic society—and within those cities there needs to be some division of labor. That means there has to be enough extra food so that some people can build houses, some can bake bread, some can weave cloth, and so on. That, in turn, means there has to be a permanent agricultural system, with technology, tools, and irrigation systems, something that started around 10,000 years ago. Then comes trade, writing, and public monuments. However, not all acknowledged civilizations tick all the boxes; like the Incas, for example, who didn’t write. Then there’s Gobekli Tepe: a massive arrangement of T-shaped carved monoliths set in complex spiral patterns that has left scholars going back to the drawing board, It predates anything in Mesopotamia—not by a few years or even centuries—but by five and a half millennia. 52 1. The primary purpose of the passage is to: ‘A. compare the historically agreed elements of civilization to primitive societies. B. describe the various factors which usually define a civilization. C. discuss the limitations of our current understanding of what a civilization is. D. introduce a confounding factor which undermines theories on civilization, 2. According to the passage, the significance of Gobekli Tepe is that: AA. its the oldest man-made thing archeologists have ever found, B. it was built in Turkey, not in Mesopotamia. C. it predates permanent agriculture by at least 1000 years. D. itis older than the Mesopotamian civilization 3. As itis used in this passage, acknowledged most nearly means: A. confirmed. B. accepted. C. approved D. accredited, Read the following two passages. Passage 1 Interviewer: The Air Force Office of Scientific Research funds various projects, not least in medical research. Dr. Kowolski, can you give us an example? Dr. Kowlowski: Sure, well, one of the latest bits of research we've been involved in is developing synthetic red blood cells. Interviewer: Those are the one responsible for oxygen transport, right? And they contain iron, Dr. Kowlowski: That's right. Interviewer: And you would need synthetic ones... for what? If we ran out of blood for transfusions? In operations... that kind of thing? Dr. Kowlowski: No. Blood contains all kinds of things, not just hemoglobin. But since it carries ‘oxygen around the body, we figured it could transport other things too—like medicine. We're only at the trial stages, but we've managed to make synthetic red blood cells that can deliver an anticancer drug and a toxin sensor. It means that in the future, medicines could be delivered much more efficiently... they'd get straight to where they're needed, so we'd need smaller quantities and they'd work faster. 53 Passage 2 Researchers in America have made “super” red blood cells (RBCs) with all the properties of natural RBCs, and the potential to act as carrier cells for therapeutic medicines. RBCs are formed of iron rich hemoglobin, which transports oxygen around the body, but they are coated in a protein which allows them to survive longer than most cells (around 120 days), and are especially flexible to allow ‘them to squeeze through narrow capillaries and bounce back into shape when they reach larger blood vessels. The researchers made their super RBCs by coating donated blood cells with silica ‘They then added positive and negative coatings of a polymer before removing the silica (giving the cell flexibility), and then added a membrane to allow it to move in the same way as a regular RBC. ‘The cells were loaded with various cargoes, such as anti-cancer drugs and magnetic nanoparticles, and survived for 48 hours, demonstrating their capacity to deliver either medicines or toxin sensors around the body. 4. Itis implied by the interview that the medical research undertaken the by the Air Force Office of Scientific Research A. isa significant proportion of its work. B. is a small part of what it does overall C. is restricted to synthetic blood. D. is usually classified. 5. Dr Kowlowski implies that delivering medicine through synthetic super RBCs would cost less because: ‘A. as the process becomes routine, there are economies of scale, so making super RBCs would cost less per unit B, less medicine is required overall as delivery is direct and there is no waste as it goes straight into the bloodstream. C. RBCs are very small, so they deliver minute amounts of medicine just where itis needed. D. they work far more quickly than traditional medicines, reducing the likelihood of over- medication. 6. Given RBCs usually last 120 days, how does the author of Passage 2 probably view the life of 48 hours for the synthetic “super” RBCs? ‘A. With caution: they should last longer than regular RBCs. 8. With concern: the blood cells would die after just 2 days. C. With interest: they already work—they just need to last longer. D. With optimism: even short-lived super RBCs can deliver a payload. 34 7, Which choice best describes the relationship between Passage 1 and Passage 2? ‘A. Passage 2 offers an evaluation of the significance of the research discussed in Passage 1, B, Passage 2 suggests an alternative use to the advancement explained in Passage 1. C. Passage 2 gives explicit details of how to achieve the outcome referred to in Passage 1 D, Passage 2 takes a more cautious approach to the findings mentioned in Passage 1. Read the following passage. NASA has been measuring winter sea ice in the Arctic and has discovered a strange phenomenon. Despite glacier loss and melting ice caps, winter sea ice is on the rise. It seems paradoxical, but NASA warns that it doesn’t mean global warming isn’t happening, only that Nature is somehow mitigating the loss of polar ice and melting glaciers by shoring up the thin sea ice of the Arctic winter. In the 1980s, sea ice in the Arctic at the start of winter (October) was over 6 feet in depth. Over the winter it would thicken, adding another three feet to the ice by spring, Last October the ice started at 3 feet but grew by 5 feet. Overall, it is still a foot thinner than forty years ago, but ‘the damage is less severe than it could have been. Itis significant because cold, fresh water sinks, and warm salty water rises. As the oceans are diluted by melting ice, the convection currents rising from the tropics won't flow as far north, and the northern hemisphere will cool. That's precisely what happened in the Little Ice Age which reached its peak in the late 17 century and resulted in widespread famine across Europe. 8. Which choice characterizes NASA's explanation for the changing pattern of Arctic sea ice? A. Itis @ natural phenomenon, with the ice thicker in some years and thinner in others. B. Thicker winter sea ice acts as a partial counterbalance to global warming. C. Nature compensates for ice loss in autumn by laying down extra ice during the winter. D. The sea ice in the Arctic is now thicker because, overall, the Earth is cooling down. 9. According to the passage, ifthe trend since the 1980s continues, the likely long-term result will be: A. even thicker winter sea ice. B, a reversal in sea currents. C. another Ice Age. D. unknown, 55. 10. What does the author mean by the claim that the oceans will be “diluted”? ‘A. They will contain more water. 8. They will contain less heat. C. They will contain more ice. D. They will contain less salt. 56 ANSWERS Informational Text - Advanced Quiz 2 The main purpose of the passage is to introduce some new evidence which turns existing theory on its head. The theory was that monuments are built by societies who go through certain stages in becoming a civilization—including having sufficiently sophisticated permanent farming in order to support a skilled labor force able to create monuments. Gobekli Tepe is a monument built by a society before permanent farming began, which is a “confounding factor.” (A) is incorrect because the passage doesn’t compare primitive societies to civlizations—it just lists the usual factors sociologists understand as characteristic of a civilization. (B) is incorrect because the passage goes further than just describing those factors—it introduces an ancient site that doesn’t conform. (C) is incorrect because the passage is not about the limitations of our current understanding, but a piece of evidence that overthrows accepted theory. The important factor is that in order to spend time building monuments, somebody has to supply the labor force with food, which means farming has to be permanent and above subsistence level. If that happened 10,000 years ago, but Gobekli Tepe predates Mesopotamia by 5,500 years (and that was in 3,500 BC or 5,500 years ago) then Gobekli Tepe was built 11,000 years ago—1000 years before permanent farming is supposed to have happened. (A) is incorrect because man-made things go back millions of years. (8) is incorrect because the location is not the important point. (0) is correct—but incomplete. The significance of Gobekli Tepe is not that it predates Mesopotamia, but that—according to existing theories—it shouldn't exist. While acknowledged can mean all of these things, in the context of this passage it means accepted or generally agreed upon. The interviewer says: “The Air Force Office of Scientific Research funds various projects, not least in medical research,” which suggests itis an important part of its work. (B) is incorrect because a small part of what it does would not be describes as “not least.” (C) is incorrect because Dr. Kowlowski says: “one of the latest bits of research we've been involved in is developing synthetic red blood cells,” so it cannot be the only project. (D) is incorrect because the interview does not discuss what is or isn't classified. 37 Dr. Kowlowski says that red blood cells were chosen as a delivery system because they carry oxygen to every part of the body; that means that—unlike regular medicines that are often ingested and absorbed via the stomach—the medicine carried on RBCs would be delivered directly, with no loss of speed, or waste, and so much smaller amounts would be needed. (A) is incorrect because while this may be true, itis not implied by the speaker. (C) is incorrect because although each cell would deliver a very tiny amount of medicine, Dr. Kowlowski doesn’t suggest patients would be given only one cell. (D) is also likely true, if by “traditional” medicine, oral medicine is meant, but Dr. Kowlowski doesn’t compare RBC delivery systems to oral or regular intravenous medication The author says that the RBCs in the tests the researchers did “survived for 48 hours, demonstrating their capacity to deliver either medicines or toxin sensors around the body,” so he or she is optimistic about their lifespan. So long as they deliver the medicine, how long they last is irrelevant. (A), (B) and (C) are incorrect for the same The big difference between the interview and the text is that the text goes into detail about how the RBCs were made, using a silicon coating and layers of a polymer, whereas the interview concentrates on what they will do. (A) is incorrect because the text doesn’t give an evaluation of the research’ significance. (B) is partially correct, because the text says the RBCs delivered “anti-cancer drugs and magnetic nanoparticles” and Dr. Kowlowski says they delivered “an anticancer drug and a toxin sensor” but both are simply giving examples of what the cell can carry—not proposing a different use. (0) is incorrect, because the author of Passage 2 seems even more optimistic than Dr. Kowlowski The passage says, “winter sea ice is on the rise...but NASA warns that it doesn’t mean global warming isn’t happening, only that Nature is somehow mitigating the loss.” In other words, Nature is “shoring up” or providing a counterbalance for the melting ice caps—but only a partial one as the ice is still thinner that it used to be. (A) is incorrect as, although it is true, that isn’t how NASA explains it. (C)is tricky, because itis. partially true, but the compensation is not complete. (D) is a misrepresentation of the text: the earth is cooling, but the passage only refers to ocean currents and the northern hemisphere cooling, and winter sea ice is thicker, but not the ice overall 58 10. The logical conclusion is that if ice continues to melt, and the oceans become less salty, then the convection currents won't bring warm water up from the tropics, so the northern hemisphere will cool. If it keeps cooling, there will be an Ice Age— eventually—and the passage points out that: “That's precisely what happened in the Little Ice Age which reached its peak in the late 17" century.” (A) is tricky, because in an ice age, there would be thicker ice—but permanently. (B) is incorrect, because the currents wouldn't reverse, they would just contract. (D) is incorrect because the passage points us towards the conclusion that we would be headed for another Ice Age. To dilute something means to make something thinner—usually by adding water, but if we add fresh water from melting glaciers and ice caps to the ocean water, it actually reduces the salinity (saltiness), so it would therefore contain less salt. (A) is, incorrect, because although the oceans would contain more water (and sea levels would rise) the author is referring to the effect on salinity. (B) is incorrect, because although the oceans would contain less heat, that isn’t what the author means by “dilute.” (C) is incorrect because although there would be more ice, that isn’t what the author means by “ilute.” 59 ©YBtong Reading Questions for the MAP Exam® 8" Grade Informational Text - Advanced 10 Questions Quiz 3 Directions: Identify the choice that best completes the statement or answers the question. Read the following passage. Ever been told to “grow up!” by an infuriated adult? Well, seems our brains do grow up—of their own accord—making two significant changes between the ages of 14 and 24. A team from the University of Cambridge, England, has been busy examining MRI scans and has discovered that the brain is edited to improve the speed and efficiency at which it works. The first change is that the cortex, which is the outermost layer of the brain, slims down. Scientists believe this is because unused or inefficient neural connections, called synapses, are abandoned. Meanwhile, the strongest and most efficient synapses are improved by myelination. Nerve cells have long fibers called axons which connect neurons at hubs called synapses. Myelination involves wrapping axons ina plasma membrane, forming chains a bit lke a string of sausages. The length and thickness of each “sausage” determines the speed of transmission and how one signal interacts with another converging at the same synapse: myelinated axons can carry information thousands of times faster than regular axons. These changes are part of the process of developing executive function: being able to curb impulses and make logical decisions, so they are an important part of the maturation process—but one over which we have no control. 60 1. The primary purpose of the passage is to: ‘A. explain that between the ages of 14 and 24 young people have no control over their behavior as their brains are still maturing. B. describe two critical alterations within the brain that take place as part of the maturation process in people aged 14-24. C. counter the widespread belief that “growing up” is a matter of physical changes and behavioral adjustments within one’s control, D. detail the critical role of myelination in strengthening axons and furthering the development of executive function in young people. 2. The author's reference to a “chain of sausages” mainly serves to suggest the: A. plasma membrane around axons, B, length of axon chains. C. different sizes of axons. D. link-like shape of axons. 3. Which choice provides the best evidence that a number of synapses developed in childhood are of little use in adulthood? A. “the brain is edited to improve the speed and efficiency at which it works” B, “the cortex, which is the outermost layer of the brain, slims down” C. “unused or inefficient neural connections, called synapses, are abandoned” D. “myelinated axons can carry information thousands of times faster than regular axons” 61 Read the following passage and chart. Passage: Scientists look at a number of indicators when they refer to climate change. In addition to recording daily air, land, and sea temperatures, precipitation patterns, and sea levels—as you'd expect—they also consider things like changes to streams and wildfires. Since 1965, wildfires have increased in frequency (many in western states) and 90% of the most severe fires have occurred since 2000. Streams are getting warmer (by 1.4°F on average) and the peak flow from snowmelt is. around 5 days earlier than it used to be. Also, earlier than it used to be is birdsong. Many native ‘American birds now sing up to 11 days earlier than they used to, and the United States Environmental Protection Agency claims that “bird species in North America have shifted their wintering grounds northward by an average of more than 40 miles since 1966, with several species shifting by hundreds of miles.” This is because winter temperatures have become less severe further north than they used to be. In addition, many birds that used to only frequent coastal habitats are finding their way inland now that temperatures are on the rise. Climate change has. also affected when plants first come into leaf and when they begin to flower. In the south, they tend to leaf and bloom later than they used to, while in the north they do it earlier than they used to, This has an impact on food crops such as strawberries. The U.S. produces 20% of the world’s strawberries, 91% of them from California, but in recent years, production has begun shifting northwards and eastwards as weather conditions there improve, and California suffers more frequent periods of drought, Chart: The following chart was used in a presentation about the effects of climate change on birds. Northward Extension of Range for Three Species of North ‘American Songbird (Miles) (Fahrenheit) 1651s ass 1885 2008s att american Goldfinch American Robin American Blackbirs — Increase in average land surface temperature 62 4, It may be inferred from the passage that scientists regard wildfires as a climate change indicator because: ‘A. they have increased in frequency since 1965 at the same time as global temperatures have increased. B, they occur after periods of extended drought which suggests higher temperatures and lower rainfall C. the vast majority of the worst recorded wildfires occurred after 2000 when climate change became a significant issue D. aside from human causes, they are generally started by to natural weather systems. lightening, which suggests changes 5. The claim that “winter temperatures have become less severe further north than they used to be” is best supported by which other detail in the passage? A. “the peak flow from snowmelt is around 5 days earlier than it used to be” B. “American birds now sing up to 11 days earlier than they used to” C. “birds that used only to frequent coastal habitats are finding their way inland now that temperatures are on the rise” D. “in the north, plants and trees leaf and bloom earlier than they used to” 6. According to the chart, which of the following statements is FALSE? ‘A. As temperatures have risen, the distance by which the American Goldfinch has extended its range has accelerated compared to the American Blackbird, B. The American Blackbird’s territory has extended northward by the greatest extent ‘compared to the American Robin and American Goldfinch. C. The American Robin can now be found more than 40 miles further north than was the case in 1965. D. In 1995 when temperatures dipped, all three species’ northward migration was halted or significantly curtailed. 7. The speaker at the presentation would likely point to which fact in the passage as offering @ logical reason for the birds’ northward shift? A. Streams are becoming warmer. B, Snow melts sooner in the north than it did C. The south and west are becoming drier. D. Trees and plants bloom earlier in the north. 63 Read the following passage. ‘Team working and group decision-making have become the norm everywhere from elementary schools to high-tech companies, and there are many advantages to collaborative effort. Things that one mind might overlook are usually spotted by someone in the group, and the various different points of view are much more representative of a mixed society than the views of one person alone, Furthermore, every individual has different experiences, skills, and knowledge, so joint projects and group decisions are often better informed. However, despite the benefits, there are drawbacks too. Coming to agreement can be time-consuming, and if two or more people in a group have strong opinions, they may not reach agreement at all, leading to frustration or compromise. That happened with the Pontiac Aztek, launched by General Motors in 2000, which was so watered down that instead of being the great crossover SUV of the age, “the tough, cool- looking concept had been reduced to a bulky, plastic-clad mess.” According to behavioral experts, Wwe need to be careful about distinguishing between collaboration and consensus, and to understand that all voices are not equal. 8. The main idea of the passage is that: A. there are pros and cons to teamwork. 8B. compromise is inevitable in teamwork. C. someone needs to coordinate teamwork, D. collaborative efforts are more inclusive. 9. The best evidence for your answer to question 8 is: A. “joint projects and group decisions are often better informed” 8. “if... people ... have strong opinions, they may not reach agreement at all” C. “despite the benefits, there are drawbacks too” D. “we need to ... understand that all voices are not equal” 10. The author's reference to the Pontiac Aztec primarily serves to: ‘A. highlight the drawbacks of misguided teamwork. 8. offer an extreme example of the difference between consensus and collaboration C. indicate the problems that arise if several people have strong opinions. D. suggest that delays due to teamwork resulted in poorly-considered decisions. 64 ANSWERS Informational Text - Advanced Quiz 3 The main purpose of the passage is to describe the two big changes that happen in the brains of young people aged 14-24 connected with “growing up”: the cortex slimming down and the process of myelination. (A) is incorrect because the passage says that “These changes are ...an important part of the maturation process—but one over which we have no control,” but it doesn’t suggest 14-24 year-olds have no control over their behavior. (C) is incorrect because while the passage suggests that some aspects of growing up (brain maturation) are out of our hands, it doesn’t suggest they all are, so it can’t counter the widespread (parents’) belief that growing up is down to choice. (D) is incorrect because although the passage explains myelination, that is only one part of the process of “developing executive function.” The author's use of an image (a chain of sausages) is to describe the link-shapes of axons. (A), (B) and (C) are incorrect because the mental picture of a chain of sausages doesn’t immediately suggest the membrane around each sausage, the length of a chain, or the different sizes of axons, mtn get rs e~SK ies o ‘The best evidence is that if “unused or inefficient neural connections, called synapses, are abandoned” between the ages of 14-24 then, having been developed in childhood, they aren’t going to be needed in adulthood. (A) and (B) are true, and provide evidence that what was going on in the brain before the age of 14 needs to change before adulthood, but neither are as specific as choice C. (D) is incorrect because this gives evidence of what is needed as an adult, not why some synapses from childhood are little use later on in life. 65, Itis logical to infer that wildfires occur in hot, dry conditions (damp wood doesn’t burn), such as after an extended drought, which in turn suggests higher temperatures and less rainfall. (A) is incorrect because although the passage says wildfires have increased since 1965 and that scientists measure changes in temperature, the passage doesn’t say that “global temperatures have increased” over that period— that’s what the chart shows. (C) is incorrect because there is nothing to indicate that climate change began in 2000—it began long before that. (0) is incorrect because while that may be true, there is nothing in the passage to point us towards lightening and changing weather systems. If snow, which is a northern phenomenon, melts earlier than it used to, then that supports the claim that winter temperatures have become less severe—otherwise the ice would melt at the same time or even later. (B) and (C) are incorrect because birds singing 11 days earlier or moving inland, doesn’t tie up with moving north. (D) is tricky, but plants and trees leaf in spring and bloom in summer, so this is not the “pest” support for the claim ‘The chart starts in 1965, by which time the robins were already 12 miles further north than prior to 1965, meaning that by the end of the chart in 2020, they had moved a farther 29 miles, making 41 miles in total. (A), (B), and (0) are all correct. ‘The most logical reason is that trees and plants are now blooming earlier in the north, giving the birds shelter earlier than before and a supply of food earlier than before. (A) is incorrect because birds are unlikely to move north for warm streams. (8) is incorrect because birds still need to eat and find somewhere to live, so melting snow is insufficient reason to move north. (C) is true, but the passage only mentions that wildfires are happening in the west and that California has suffered increasing droughts, so that wouldn’t explain a general northward shift (east coast included). The main point the passage is trying to get across is that there are pros ("the various different points of view are much more representative of a mixed society than the Views of one person”) to teamwork as well as cons (“Coming to agreement can be time-consuming”). (8) and (0) are incorrect because although true statements, neither point is the central idea and (D) is not mentioned, (C) is incorrect because this idea is not mentioned in the passage either. ‘The best evidence that there are pros and cons to teamwork is the statement that “despite the benefits, there are drawbacks too.” (A) is incorrect because this choice would support the idea that collaborative efforts are inclusive. (8) is incorrect because this choice would support the idea that compromise is inevitable. (D) is incorrect because this choice would support the idea that someone needs to coordinate teamwork. 66

You might also like